True or False?( Calculus)












0














If $F(x)$ is a continuous function in $[0,1]$ and $F(x) = 1$ for all rational numbers then $F(1/sqrt2) = 1$. (True/ False)



I think the statement is true because since $F$ is continuous and $1/sqrt2$ is in neighbourhood of some rational number so $F(1/sqrt2)$ should be one so that limit can exist. Is this reason valid to prove the statement true.










share|cite|improve this question
























  • Yes, yes, yes, yes. (Sorry I can't only type one 'yes')
    – Seewoo Lee
    Nov 24 at 5:29










  • @SeewooLee try Yes ${}{}{}{}{}{}$
    – Chase Ryan Taylor
    Nov 24 at 5:30






  • 1




    Yes, you thinking is correct. You can improve your wording. "so F(1/2–√) should be one so that limit can exist" is badly worded and semantically ... weird. But I know what you are trying to express and it is absolutely correct.
    – fleablood
    Nov 24 at 5:30










  • Joel Pereira's answer is a very nice mathematical way to express your thinking.
    – fleablood
    Nov 24 at 5:32










  • "$1/sqrt{2}$ is in neighborhood of some rational number" is not what you want to be saying. The idea you want to convey is "every neighborhood of $1/sqrt{2}$ contains a rational number", which is what @Joel Pereira does more precisely in his answer.
    – JonathanZ
    Nov 24 at 5:40
















0














If $F(x)$ is a continuous function in $[0,1]$ and $F(x) = 1$ for all rational numbers then $F(1/sqrt2) = 1$. (True/ False)



I think the statement is true because since $F$ is continuous and $1/sqrt2$ is in neighbourhood of some rational number so $F(1/sqrt2)$ should be one so that limit can exist. Is this reason valid to prove the statement true.










share|cite|improve this question
























  • Yes, yes, yes, yes. (Sorry I can't only type one 'yes')
    – Seewoo Lee
    Nov 24 at 5:29










  • @SeewooLee try Yes ${}{}{}{}{}{}$
    – Chase Ryan Taylor
    Nov 24 at 5:30






  • 1




    Yes, you thinking is correct. You can improve your wording. "so F(1/2–√) should be one so that limit can exist" is badly worded and semantically ... weird. But I know what you are trying to express and it is absolutely correct.
    – fleablood
    Nov 24 at 5:30










  • Joel Pereira's answer is a very nice mathematical way to express your thinking.
    – fleablood
    Nov 24 at 5:32










  • "$1/sqrt{2}$ is in neighborhood of some rational number" is not what you want to be saying. The idea you want to convey is "every neighborhood of $1/sqrt{2}$ contains a rational number", which is what @Joel Pereira does more precisely in his answer.
    – JonathanZ
    Nov 24 at 5:40














0












0








0







If $F(x)$ is a continuous function in $[0,1]$ and $F(x) = 1$ for all rational numbers then $F(1/sqrt2) = 1$. (True/ False)



I think the statement is true because since $F$ is continuous and $1/sqrt2$ is in neighbourhood of some rational number so $F(1/sqrt2)$ should be one so that limit can exist. Is this reason valid to prove the statement true.










share|cite|improve this question















If $F(x)$ is a continuous function in $[0,1]$ and $F(x) = 1$ for all rational numbers then $F(1/sqrt2) = 1$. (True/ False)



I think the statement is true because since $F$ is continuous and $1/sqrt2$ is in neighbourhood of some rational number so $F(1/sqrt2)$ should be one so that limit can exist. Is this reason valid to prove the statement true.







calculus proof-verification






share|cite|improve this question















share|cite|improve this question













share|cite|improve this question




share|cite|improve this question








edited Nov 24 at 5:29









Tianlalu

3,12321038




3,12321038










asked Nov 24 at 5:22









Mathsaddict

1658




1658












  • Yes, yes, yes, yes. (Sorry I can't only type one 'yes')
    – Seewoo Lee
    Nov 24 at 5:29










  • @SeewooLee try Yes ${}{}{}{}{}{}$
    – Chase Ryan Taylor
    Nov 24 at 5:30






  • 1




    Yes, you thinking is correct. You can improve your wording. "so F(1/2–√) should be one so that limit can exist" is badly worded and semantically ... weird. But I know what you are trying to express and it is absolutely correct.
    – fleablood
    Nov 24 at 5:30










  • Joel Pereira's answer is a very nice mathematical way to express your thinking.
    – fleablood
    Nov 24 at 5:32










  • "$1/sqrt{2}$ is in neighborhood of some rational number" is not what you want to be saying. The idea you want to convey is "every neighborhood of $1/sqrt{2}$ contains a rational number", which is what @Joel Pereira does more precisely in his answer.
    – JonathanZ
    Nov 24 at 5:40


















  • Yes, yes, yes, yes. (Sorry I can't only type one 'yes')
    – Seewoo Lee
    Nov 24 at 5:29










  • @SeewooLee try Yes ${}{}{}{}{}{}$
    – Chase Ryan Taylor
    Nov 24 at 5:30






  • 1




    Yes, you thinking is correct. You can improve your wording. "so F(1/2–√) should be one so that limit can exist" is badly worded and semantically ... weird. But I know what you are trying to express and it is absolutely correct.
    – fleablood
    Nov 24 at 5:30










  • Joel Pereira's answer is a very nice mathematical way to express your thinking.
    – fleablood
    Nov 24 at 5:32










  • "$1/sqrt{2}$ is in neighborhood of some rational number" is not what you want to be saying. The idea you want to convey is "every neighborhood of $1/sqrt{2}$ contains a rational number", which is what @Joel Pereira does more precisely in his answer.
    – JonathanZ
    Nov 24 at 5:40
















Yes, yes, yes, yes. (Sorry I can't only type one 'yes')
– Seewoo Lee
Nov 24 at 5:29




Yes, yes, yes, yes. (Sorry I can't only type one 'yes')
– Seewoo Lee
Nov 24 at 5:29












@SeewooLee try Yes ${}{}{}{}{}{}$
– Chase Ryan Taylor
Nov 24 at 5:30




@SeewooLee try Yes ${}{}{}{}{}{}$
– Chase Ryan Taylor
Nov 24 at 5:30




1




1




Yes, you thinking is correct. You can improve your wording. "so F(1/2–√) should be one so that limit can exist" is badly worded and semantically ... weird. But I know what you are trying to express and it is absolutely correct.
– fleablood
Nov 24 at 5:30




Yes, you thinking is correct. You can improve your wording. "so F(1/2–√) should be one so that limit can exist" is badly worded and semantically ... weird. But I know what you are trying to express and it is absolutely correct.
– fleablood
Nov 24 at 5:30












Joel Pereira's answer is a very nice mathematical way to express your thinking.
– fleablood
Nov 24 at 5:32




Joel Pereira's answer is a very nice mathematical way to express your thinking.
– fleablood
Nov 24 at 5:32












"$1/sqrt{2}$ is in neighborhood of some rational number" is not what you want to be saying. The idea you want to convey is "every neighborhood of $1/sqrt{2}$ contains a rational number", which is what @Joel Pereira does more precisely in his answer.
– JonathanZ
Nov 24 at 5:40




"$1/sqrt{2}$ is in neighborhood of some rational number" is not what you want to be saying. The idea you want to convey is "every neighborhood of $1/sqrt{2}$ contains a rational number", which is what @Joel Pereira does more precisely in his answer.
– JonathanZ
Nov 24 at 5:40










2 Answers
2






active

oldest

votes


















2














For every $n$, there exists a rational number $q_n$ such that $mid frac{1}{sqrt{2}}-q_nmid < frac{1}{n}$. So $displaystylelim_{nrightarrow infty}q_n$ = $frac{1}{sqrt{2}}$. Since F is continuous, 1 = $displaystylelim_{nrightarrow infty}$F($q_n$) = F($frac{1}{sqrt{2}}$) .






share|cite|improve this answer





























    1














    The statement is true, although I would express it: there is a sequence ${r_n}_{ninmathbb{N}}$ of rational numbers in the interval $[0,1]$ such that $r_n to 1/sqrt{2}$, hence by continuity
    $$
    F(1/sqrt{2}) =
    F(lim_{ntoinfty} r_n) =
    lim_{ntoinfty} F(r_n) =
    lim_{ntoinfty} 1 = 1 ,.
    $$






    share|cite|improve this answer





















      Your Answer





      StackExchange.ifUsing("editor", function () {
      return StackExchange.using("mathjaxEditing", function () {
      StackExchange.MarkdownEditor.creationCallbacks.add(function (editor, postfix) {
      StackExchange.mathjaxEditing.prepareWmdForMathJax(editor, postfix, [["$", "$"], ["\\(","\\)"]]);
      });
      });
      }, "mathjax-editing");

      StackExchange.ready(function() {
      var channelOptions = {
      tags: "".split(" "),
      id: "69"
      };
      initTagRenderer("".split(" "), "".split(" "), channelOptions);

      StackExchange.using("externalEditor", function() {
      // Have to fire editor after snippets, if snippets enabled
      if (StackExchange.settings.snippets.snippetsEnabled) {
      StackExchange.using("snippets", function() {
      createEditor();
      });
      }
      else {
      createEditor();
      }
      });

      function createEditor() {
      StackExchange.prepareEditor({
      heartbeatType: 'answer',
      autoActivateHeartbeat: false,
      convertImagesToLinks: true,
      noModals: true,
      showLowRepImageUploadWarning: true,
      reputationToPostImages: 10,
      bindNavPrevention: true,
      postfix: "",
      imageUploader: {
      brandingHtml: "Powered by u003ca class="icon-imgur-white" href="https://imgur.com/"u003eu003c/au003e",
      contentPolicyHtml: "User contributions licensed under u003ca href="https://creativecommons.org/licenses/by-sa/3.0/"u003ecc by-sa 3.0 with attribution requiredu003c/au003e u003ca href="https://stackoverflow.com/legal/content-policy"u003e(content policy)u003c/au003e",
      allowUrls: true
      },
      noCode: true, onDemand: true,
      discardSelector: ".discard-answer"
      ,immediatelyShowMarkdownHelp:true
      });


      }
      });














      draft saved

      draft discarded


















      StackExchange.ready(
      function () {
      StackExchange.openid.initPostLogin('.new-post-login', 'https%3a%2f%2fmath.stackexchange.com%2fquestions%2f3011216%2ftrue-or-false-calculus%23new-answer', 'question_page');
      }
      );

      Post as a guest















      Required, but never shown

























      2 Answers
      2






      active

      oldest

      votes








      2 Answers
      2






      active

      oldest

      votes









      active

      oldest

      votes






      active

      oldest

      votes









      2














      For every $n$, there exists a rational number $q_n$ such that $mid frac{1}{sqrt{2}}-q_nmid < frac{1}{n}$. So $displaystylelim_{nrightarrow infty}q_n$ = $frac{1}{sqrt{2}}$. Since F is continuous, 1 = $displaystylelim_{nrightarrow infty}$F($q_n$) = F($frac{1}{sqrt{2}}$) .






      share|cite|improve this answer


























        2














        For every $n$, there exists a rational number $q_n$ such that $mid frac{1}{sqrt{2}}-q_nmid < frac{1}{n}$. So $displaystylelim_{nrightarrow infty}q_n$ = $frac{1}{sqrt{2}}$. Since F is continuous, 1 = $displaystylelim_{nrightarrow infty}$F($q_n$) = F($frac{1}{sqrt{2}}$) .






        share|cite|improve this answer
























          2












          2








          2






          For every $n$, there exists a rational number $q_n$ such that $mid frac{1}{sqrt{2}}-q_nmid < frac{1}{n}$. So $displaystylelim_{nrightarrow infty}q_n$ = $frac{1}{sqrt{2}}$. Since F is continuous, 1 = $displaystylelim_{nrightarrow infty}$F($q_n$) = F($frac{1}{sqrt{2}}$) .






          share|cite|improve this answer












          For every $n$, there exists a rational number $q_n$ such that $mid frac{1}{sqrt{2}}-q_nmid < frac{1}{n}$. So $displaystylelim_{nrightarrow infty}q_n$ = $frac{1}{sqrt{2}}$. Since F is continuous, 1 = $displaystylelim_{nrightarrow infty}$F($q_n$) = F($frac{1}{sqrt{2}}$) .







          share|cite|improve this answer












          share|cite|improve this answer



          share|cite|improve this answer










          answered Nov 24 at 5:30









          Joel Pereira

          65119




          65119























              1














              The statement is true, although I would express it: there is a sequence ${r_n}_{ninmathbb{N}}$ of rational numbers in the interval $[0,1]$ such that $r_n to 1/sqrt{2}$, hence by continuity
              $$
              F(1/sqrt{2}) =
              F(lim_{ntoinfty} r_n) =
              lim_{ntoinfty} F(r_n) =
              lim_{ntoinfty} 1 = 1 ,.
              $$






              share|cite|improve this answer


























                1














                The statement is true, although I would express it: there is a sequence ${r_n}_{ninmathbb{N}}$ of rational numbers in the interval $[0,1]$ such that $r_n to 1/sqrt{2}$, hence by continuity
                $$
                F(1/sqrt{2}) =
                F(lim_{ntoinfty} r_n) =
                lim_{ntoinfty} F(r_n) =
                lim_{ntoinfty} 1 = 1 ,.
                $$






                share|cite|improve this answer
























                  1












                  1








                  1






                  The statement is true, although I would express it: there is a sequence ${r_n}_{ninmathbb{N}}$ of rational numbers in the interval $[0,1]$ such that $r_n to 1/sqrt{2}$, hence by continuity
                  $$
                  F(1/sqrt{2}) =
                  F(lim_{ntoinfty} r_n) =
                  lim_{ntoinfty} F(r_n) =
                  lim_{ntoinfty} 1 = 1 ,.
                  $$






                  share|cite|improve this answer












                  The statement is true, although I would express it: there is a sequence ${r_n}_{ninmathbb{N}}$ of rational numbers in the interval $[0,1]$ such that $r_n to 1/sqrt{2}$, hence by continuity
                  $$
                  F(1/sqrt{2}) =
                  F(lim_{ntoinfty} r_n) =
                  lim_{ntoinfty} F(r_n) =
                  lim_{ntoinfty} 1 = 1 ,.
                  $$







                  share|cite|improve this answer












                  share|cite|improve this answer



                  share|cite|improve this answer










                  answered Nov 24 at 5:42









                  mlerma54

                  1,087138




                  1,087138






























                      draft saved

                      draft discarded




















































                      Thanks for contributing an answer to Mathematics Stack Exchange!


                      • Please be sure to answer the question. Provide details and share your research!

                      But avoid



                      • Asking for help, clarification, or responding to other answers.

                      • Making statements based on opinion; back them up with references or personal experience.


                      Use MathJax to format equations. MathJax reference.


                      To learn more, see our tips on writing great answers.





                      Some of your past answers have not been well-received, and you're in danger of being blocked from answering.


                      Please pay close attention to the following guidance:


                      • Please be sure to answer the question. Provide details and share your research!

                      But avoid



                      • Asking for help, clarification, or responding to other answers.

                      • Making statements based on opinion; back them up with references or personal experience.


                      To learn more, see our tips on writing great answers.




                      draft saved


                      draft discarded














                      StackExchange.ready(
                      function () {
                      StackExchange.openid.initPostLogin('.new-post-login', 'https%3a%2f%2fmath.stackexchange.com%2fquestions%2f3011216%2ftrue-or-false-calculus%23new-answer', 'question_page');
                      }
                      );

                      Post as a guest















                      Required, but never shown





















































                      Required, but never shown














                      Required, but never shown












                      Required, but never shown







                      Required, but never shown

































                      Required, but never shown














                      Required, but never shown












                      Required, but never shown







                      Required, but never shown







                      Popular posts from this blog

                      Plaza Victoria

                      In PowerPoint, is there a keyboard shortcut for bulleted / numbered list?

                      How to put 3 figures in Latex with 2 figures side by side and 1 below these side by side images but in...